Describe the story of the constellation Capricornus. WILL MARK BRAINLEST IF NOT STOLEN FROM A WEBSITE OR ANYTHING WITH A PROPER ANSWER ASSAP

Answers

Answer 1

Your answer:

In Greek mythology, this constellation is related with the time the Olympian gods sought refuge in Egypt. Unfortunately, following their epic fighting with the Titans, peace did not closing for long, as the monster Typhon, son of the Titan Tartarus and Earth, sought revenge. Typhon was once a fearsome fire-breathing creature, taller than mountains and with palms which possessed dragons' heads in region of fingers. The Olympian gods sought to break out by way of adopting a number disguises: Zeus, a ram - Hera, a white cow, Bacchus (another model of the fable suggests Pan) a goat. As Typhon approached, Bacchus/Pan threw himself into the Nile but, in a panic, solely succeeded in altering part of his body, ending up with a goat's physique and the tail of a fish. Meanwhile, Zeus had been dismembered via Typhon, however was saved when Bacchus/Pan let out an ear-splitting yell, distracting the monster lengthy ample for an agile Hermes to gather the supreme god's limbs and cautiously fix him. In gratitude, Zeus transferred Bacchus/Pan to the heavens.

Answer 2

Answer:

Located in the Southern Hemisphere, Capricornus represents a creature that is a blend of fish and goat; the name means "goat horn" in Latin. While it is one of the Zodiac constellations identified by the Greek astronomer Ptolemy in the 2nd century, Capricornus is often used when referring to the constellation, while Capricorn is used in conjunction with the sign of the Zodiac.Locating Capricornus

Capricornus is among the faintest constellations, just brighter than Cancer. The 40th largest constellation, Capricornus measures 414 square degrees. It sits among the other constellations of the water genre, including the water-bearer Aquarius, the whale-like sea monster Cetus, the fishes Pisces, and the river Eridanus.

Right Ascension: 21.02 hours

Declination: -20 to -23 degrees

Visible: between latitudes 60 degrees and -90 degrees

Best viewed: During the month of September at 9 p.m.

To find the arrowhead-shaped Capricornus in the sky, look for the Summer Triangle and make a line from Vega through Altair to the lower southern sky.

The tropic of Capricorn the place where the sun appears overhead at noon on the winter solstice originally sliced right through its namesake constellation, but the line has since shifted to go through the constellation .

The constellation has its roots in the cultures of Sumeria — which identified it with a mythical figure that was half goat and half fish — and Babylonia, which portrayed it as a goat-human hybrid.

The Greeks associate the constellation with Pan, the god of nature. Part of Pan's lore was that he helped Zeus fight the Titans to earn his spot in the heavens. He escaped the monster Typhon by jumping into the Nile, but only half of his body was submerged, so he was a fish in the part of his body that remained underwater. Other spins on the tale have the constellation associated with Amalthaea, the mythical goat that acted as a foster mother to Zeus as an infant.


Related Questions

Two wires, both with current out of the page, are next to one another. The wire on the left has a current of 1 A and the wire on the right has a current of 2 A. We can say that:

A. The left wire attracts the right wire and exerts twice the force as the right wire does.
B. The left wire attracts the right wire and exerts half the force as the right wire does.
C. The left wire attracts the right wire and exerts as much force as the right wire does.
D. The left wire repels the right wire and exerts twice the force as the right wire does.
E. The left wire repels the right wire and exerts half the force as the right wire does.
F. The left wire repels the right wire and exerts as much force as the right wire does.

Answers

Answer:

C. The left wire attracts the right wire and exerts as much force as the right wire does.

Explanation:

To know what is the answer you first take into account the magnetic field generated by each current, for a distance of d:

[tex]B_1=\frac{\mu_oI_1}{2\pi d}=\frac{\mu_o}{2\pi d}(1A)\\\\B_2=\frac{\mu_oI_2}{2\pi d}=\frac{\mu_o}{2\pi d}(2A)=2B_1\\\\[/tex]

Next, you use the formula for the magnetic force produced by the wires:

[tex]\vec{F_B}=I\vec{L}\ X \vec{B}[/tex]

if the direction of the L vector is in +k direction, the first wire produced a magnetic field with direction +y, that is, +j and the second wire produced magnetic field with direction -y, that is, -j (this because the direction of the magnetic field is obtained by suing the right hand rule). Hence, the direction of the magnetic force on each wire, produced by the other one is:

[tex]\vec{F_{B1}}=I_1L\hat{k}\ X\ B_2(-\hat{j})=I_1LB_2\hat{i}=(2A^2)\frac{L\mu_o}{2\pi d}\hat{i}\\\\\vec{F_{B2}}=I_2L\hat{k}\ X\ B_2(\hat{j})=I_2LB_1\hat{i}=-(2A^2)\frac{L\mu_o}{2\pi d}\hat{i}[/tex]

Hence, due to this result you have that:

C. The left wire attracts the right wire and exerts as much force as the right wire does.

In this circuit the battery provides 3 V, the resistance R1 is 7 Ω, and R2 is 5 Ω. What is the current through resistor R2? Give your answer in units of Amps. An Amp is 1 Coulomb of charge flowing through a cross-sectional area of the wire per second - that's a lot of charge per second and will warm up a typical wire quite a bit! Most devices have circuits with larger resistors - kLaTeX: \OmegaΩ (103 LaTeX: \OmegaΩ) and MLaTeX: \OmegaΩ (106 LaTeX: \OmegaΩ) are common.

Answers

Answer:

The current pass the [tex]R_2[/tex] is  [tex]I = 0.25 A[/tex]

Explanation:

The diagram for this question is shown on the first uploaded image  

From the question we are told that

    The voltage  is  [tex]V = 3V[/tex]

     The first resistance is  [tex]R_1 = 7 \Omega[/tex]

     The second resistance is  [tex]R_2 = 5 \Omega[/tex]

Since the resistors are connected in series their equivalent resistance is  

       [tex]R_{eq} = R_1 +R_2[/tex]

Substituting values

         [tex]R_{eq} = 7 + 5[/tex]

         [tex]R_{eq} = 12 \Omega[/tex]

Since the resistance are connected in serie the current passing through the circuit  is the same current passing through [tex]R_2[/tex] which is mathematically evaluated as

        [tex]I = \frac{V}{R_{eq}}[/tex]

Substituting values  

      [tex]I = \frac{3}{12}[/tex]

      [tex]I = 0.25 A[/tex]

Newton's law of cooling states that the temperature of an object changes at a rate proportional to the difference between its temperature and that of its surroundings. Suppose that the temperature of a cup of coffee obeys Newton's law of cooling. If the coffee has a temperature of 210 degrees Fahrenheit when freshly poured, and 2.5 minutes later has cooled to 191 degrees in a room at 64 degrees, determine when the coffee reaches a temperature of 156 degrees.

Answers

Answer:

Explanation:

The problem is based on Newton's law of cooling .

According to Newton's law

dQ / dt = k ( T - T₀ ) ,

dT / dt = k' ( T - T₀ )          ; dT / dt is rate of fall of temperature.

T is average  temperature of hot body , T₀ is temperature of surrounding .

In the first case rate of fall of temperature = (210 - 191) / 2.5

= 7.6 degree / s

average temperature T = (210 + 191) /2

= 200.5  

Putting in the equation

7.6 = k' ( 200.5  - 64 )

k' = 7.6 / 136.5

= .055677

In the second case :---

In the second case, rate of fall of temperature = (191 - 156) / t  

= 35 / t   , t is time required.

average temperature T = (156 + 191) /2

= 173.5  

Putting in the equation

35 / t = .05567 ( 173.5 - 64 )

t = 5.74 minute .

Dual Nature of Light
Assignment
Active
Explaining the Nature of Light
Why do scientists believe that light is made of streams of
particles?

Answers

scientists believe that light is made of streams of particles because only the presence of photons can explain phenomena observed during experiments on the photoelectric effect.

What happens if you move a magnet near a could of wire

Answers

Answer:

The wire would stick to the magnet????????????????????????

Explanation:

A pipe branches symmetrically into two legs of length L, and the whole system rotates with angular speed ω around its axis of symmetry. Each branch is inclined at angle α to the axis of rotation. Liquid enters the pipe steadily, with zero angular momentum, at volume flow rate Q. The pipe diameter, D, is much smaller than L. Obtain an expression for the external torque required to turn the pipe. What additional torque would be required to impart angular acceleration ω_ ?

Answers

Answer:

Check the explanation

Explanation:

Kindly check the attached images below to see the step by step explanation to the question above.

Air is matter which backs best support the statement

Answers

Answer: A. Balloons can be filled with air.

C. Air has mass.

Explanation:

Learn more https://brainly.com/question/3238218

Balloons are able to be filled with air and air has mass.

A brick is dropped from a high scaffold. a. What is its velocity after 4.0s ?

b. How far does the brick fall during this time ?

Answers

Answer:

A: 1.962

B: 3.924

Explanation:

g = G *M /R^2

g = 9.807*M/R^2 the gravitational constant of ground level on earth is about 9.807

g = 9.807*5lbs/R^2 the average brick is about 5 pounds.

g = 9.807*5*10^2.   I'm assuming the height is around ten feet to help you out.

with these numbers plugged in you get an acceleration of 0.4905 a final velocity after 4 seconds 1.962. It's height fallen after 4 seconds is 3.924.

( M = whatever the brick weighs it's not specified in the question)

(R = the distance from the ground or how high the scaffold is)

(hopefully you can just plug your numbers in there hope this helps)

Is mercury (the planet) rocky or gaseous(meaning relating to or having the characteristics of a gas.)

Answers

Answer:

Mercury is rocky

Explanation:

Answer:

Rocky

Explanation:

It has no atmosphere so it cannot hold gas.

Why does current flow in a coil when a magnet is pushed in and out of the coil ?

Answers

Answer:

So the induced current opposes the motion that induced it (from Lenz's Law). When we pull the magnet out, the left hand end of the coil becomes a south pole (to try and hold the magnet back). Therefore the induced current must be flowing clockwise.

hope this helps u...


What is an independent variable?
A. A variable that is intentionally changed during an experiment
B. A variable that depends on the experimental variable
C. A variable that is not used in an experiment
D. A variable that is unknown during the experiment

Answers

Answer:

The answer is A

Explanation:

Independent variables don't have to depend on other factors of the experiment because they're independent

Answer:

A.

Explanation:

Independent variables don't have to depend on other factors of the experiment because they're independent.

Which is a characteristic of diatoms?
A)whip-like tail
B)heterotrophic
C)multicellular
D)glass-ive cell wall

Answers

Answer:

b

Explanation:

Answer:

The correct answer is D, diatoms have glass-like cell walls.

An athlete is working out in the weight room. He steadily holds 50 kilograms above his head for 10 seconds. Which statement is true about this situation?

Answers

Answer:

Answer: the true statement form the given statements is “the athletes is not doing any work because he does not move weight”

Explanation:

The athlete isn’t doing any work because he doesn’t move the weight is the correct statement.

What is Work? Work is the energy transferred to or from an object via the application of force along a displacement.Work = Force x Displacement.

How to solve this Problem?The weight of an object given is 50kgsThe time of holding an object given is 10 secondsWe need to justify the statements

Here ,

There is no displacement that means displacement is zero.If displacement is zero then work done will also be zero

Hence there is no work done by the athlete

Therefore ,The athlete isn’t doing any work because he doesn’t move the weight is the correct statement

Learn more about Work done here

https://brainly.com/question/25573309

#SPJ2

A particular coil has 100 turns and a diameter of 6.0 m. When it's time for a measurement, a 4.5 A current is turned on. The large diameter of the coil means that the field in the water flowing directly above the center of the coil is approximately equal to the field in the center of the coil. The field is directed downward and the water is flowing east. The water is flowing above the center of the coil at 1.5 m/s .

What is the magnitude of the field at the center of the coil?

Answers

Answer:

The magnetic field at the center of the coil = 5.23 * 10 ^ -5 T

Explanation:

Information from the question:

Number of turns of the coil = 100 turns

The diameter of the coil = 6 m

The radius of the coil = diameter / 2 = 3 m

The coil current = 2.5 A

Formula : The Magnetic field at the center of the coil =

                                  k * number of turns * current / 2 * radius

Therefore, The Magnetic field at the center of the coil=

                                 (4 * [tex]\pi[/tex] * 10 ^ -7 * 100 * 2.5 ) / (2 * 3)

The Magnetic field at the center of the coil = 5.23 * 10 ^ -5 T

The main component of all computer memory is

Answers

Hi!

The main component of all computer memory is RAM.

Hope this helps !

Answer: R.A.M

Explanation:

Find the frequency of the 4th harmonic waves on a violin string that is 48.0cm long with a mass of 0.300 grams
and is under a tension of 4.00N. ​

Answers

Answer:

The frequency of the 4th harmonic of the string is 481.13 Hz.

Explanation:

When a stretch string fixed at both ends is set into vibration, it produces its lowest sound of possible note called the fundamental frequency.  Under certain conditions on the string, higher frequencies called harmonics or overtones can be produced.

The frequency of the forth harmonic is the third overtone of the string and can be determined by:

          f = [tex]\frac{2}{L}[/tex][tex]\sqrt{\frac{T}{m} }[/tex]

Given that; L = 48.0 cm = 0.48 m,

                 m = 0.3 g = 0.0003 Kg,

                 T = 4.0 N,

         f = [tex]\frac{2}{0.48}[/tex][tex]\sqrt{\frac{4}{0.0003} }[/tex]

         f = 4.1667 × 115.4701

           = 481.1252

        f = 481.13 Hz

The frequency of the 4th harmonic of the string is 481.13 Hz.

I need some help!!!!!!!!!

Answers

Answer:

The Object will immediately begin moving toward the left

Explanation:

Because the force of thirteen is greater than ten and applied to the opposite side

Eclipses of the sun and moon are best explained by which of the following?

The irregular shape of the orbits of Earth and the moon


The rotation of Earth and the moon on their axis


The revolution of Earth and the moon around the sun

The different chemical compositions of the sun and the moon

Answers

Answer:

Eclipses of the Sun and Moon are best explained by the revolution of Earth and the Moon around the Sun.

Explanation:

An eclipse is the covering of a star by another celestial body.  In everyday language, an eclipse usually means a solar or lunar eclipse.

When the sun, moon and earth are aligned (in this order), the sun is covered by the moon when viewed from a place on earth. The sunlight will then no longer reach the earth. In other words, part of the earth has been eclipsed by the moon. This is referred to as a solar eclipse.

If the earth is between the sun and the moon, and thereby prevents sunlight from reaching the moon, then there is a lunar eclipse. After all, seen from the moon, the earth covers the sun. The moon is then eclipsed by the earth.

Exercise should challenge your body and be at a greater intensity than your usual bif daily activity. Discuss

Answers

Answer:

A fan pushes hot air out of a vent and into a room. The hot air displaces cold air in the room, causing the cold air to move closer to the floor.

The hot air displacing the cold air is an example of  transfer by

Explanation:

a girl pushes an 18.15 kg wagon with a force of 3.63 N. what is the acceleration?

A. 0.06 m/s2
B. 5 m/s2
C. 0.2 m/s2

Answers

Answer:0.2 m/s^2

Explanation:

mass=18.15kg

Force=3.63N

Acceleration=force ➗ mass

Acceleration=3.63 ➗ 18.15

Acceleration=0.2 m/s^2

A girl pushes a wagon of mass 18.15 kg with a force of 3.63 N, so the acceleration of the wagon will be 0.2 m/s².

What is acceleration?

In mechanics, acceleration is the measure of how rapidly an object's velocity changes over time. Accelerations as a vector quantity. An object's acceleration depends on the direction of the net force exerted on it.

A vector quantity, acceleration, is something that has both a magnitude and a direction. As a vector quantity, velocity is also. The ratio of the velocity vector change over a time interval to that interval is the definition of acceleration.

Mass, m =18.15 kg

Force, f = 3.63 N

Force = m × a

a= f / m

a =3.63 / 18.15

a = 0.2 m/s²

Hence, the acceleration of the wagon will be 0.2 m/s².

To get more information about acceleration :

https://brainly.com/question/12550364

#SPJ2

plzzz help will mark the brainliest

Answers

Ciara is winging....etc
The answer is : 0.60 N, toward the center of the circle


A satellite....etc
The Answer is : 7400 m/s


What is the .....etc
The Answer is : 2.60 m/s

"It is impossible to devise a process which may convert heat, extracted from a single
reservoir, entirely into work without leaving any change in the working system”.
Use the Second Law of Thermodynamics to state our inability to utilize the heat contents
of oceans and atmosphere which contains a large amount of heat energy but cannot be
converted into useful mechanical work You may use the concept of heat engine to discuss
this​

Answers

Answer:

According to the second law of thermodynamics, we are unable to use the heat of the ocean and the atmosphere because we do not have a reservoir that has a temperature lower than the ocean or the atmosphere.

Explanation:

As you already know, the ocean and atmosphere have a lot of thermal energy, however, we are unable to convert this energy into mechanical energy that would be useful for our activities. This can be explained by the second law of thermodynamics, since it states that the presence of two bodies with different temperatures is necessary for it to be possible to transform heat into work.

In this case, to transform the thermal energy of the ocean and the atmosphere into mechanical energy we would need the existence of a thermal motor, which is only possible to be established when there is a body with high thermal energy and a sink, a reservoir, with low thermal energy, which will be the place where the heat will be expelled, to be converted into work. We do not have a reservoir with less thermal energy than the ocean and the atmosphere, so we cannot use their energy.

The universal law of gravitation states that the force of attraction between two objects depends on which quantities?
the masses of the objects and their densities
the distance between the objects and their shapes
the densities of the objects and their shapes
the masses of the objects and the distance between them
Save and Exit
Next
Subm
Kandretum

Answers

Answer:depends on the masses of the objects and the distance between them

Explanation:

According to Newton's law of universal gravitation,the force of attraction between two objects depends on the masses of the objects and the distance between them

Question 7
Review
Which particles are not affected by the strong force?
A.
hadrons
B.
protons
C.
neutrons
D
electrons
Submit A
Hide Toolbar

Answers

Answer:

Electron

Explanation:

Because electron are not hadrons so electron are not affected by strong force

Particles that can not be affected by strong forces are electrons.

What are electrons?

Electrons are the rotating material around the nucleus of an atomic element in orbit.

Atoms have electrostatic energy between their electrons. This force is not broken by a force as strong as nuclear power.

What are strong forces?

Strong force is a fundamental interaction of nature that acts between subatomic particles of matter.

There are four basic forces in nature:

Gravity: the gravitational force used between any heavy objects. It has an infinite range.Electrical energy: energy used between electrically charged objects. It can be either attraction or repulsion.Nuclear power: is the magnetic field that responds to the binding of protons and neutrons within the nucleus of an atom. It only works for very short distances.Weak nuclear power: a force that causes nuclear decay. It only works for very short distances.

Therefore, particles that are unaffected by strong force are electrons.

To learn more about strong force here

https://brainly.com/question/9910823

#SPJ2

1. You are playing with a jump rope that is tied at both ends. You untie one end, hold it taut and wiggle the end up and down sinusoidally with frequency 2.00Hz and amplitude 0.075m. At time t=0, the end has a maximum positive displacement and is instantaneously at rest. Assume no wave bounces back from the far end to change the pattern. What is the equation for the displacement of the wave? What is the displacement at a point 3.00m from the end .

Answers

Answer:

[tex]f(x=3.00m)=0.075mcos(\frac{2\pi(2.00Hz)}{v}(3.00m))[/tex]

Explanation:

To find the equation of the wave you use the general equation for a wave, given by:

[tex]f(x)=Acos(k x-\omega t)[/tex]

A: amplitude of the wave = 0.075m

k: wave number

you select a cosine function because for x=0 and t= 0 you get a maximum displacement.

To find the displacement of the wave for x=0 you can consider that the form of the wave is independent of time t.

Then, you calculate k:

[tex]k=\frac{\omega}{v}=\frac{2\pi f}{v}[/tex]

Thus, you need the value of the speed of the wave (you only have the frequency f), in order to calculate f(x), for x=3.00m:

[tex]f(x=3.00m)=0.075mcos(\frac{2\pi(2.00Hz)}{v}(3.00m))[/tex]


1) Calculate the equivalent resistance of 2 resistors wired in parallel. The value of the resistors are
4 ohms and 8 ohms.

Answers

Explanation:

[tex]r = \: ( { \frac{4 \times 8}{4 + 8} } \\ r= \: ohm[/tex]2.67

Answer:2 2/3 ohms

Explanation:

equivalent resistance=R

r1=4 ohms

r2=8 ohms

For parallel connection

1/R=1/(r1) + 1/(r2)

1/R=1/4 + 1/8

1/R=(2x1+1x1)/8

1/R=(2+1)/8

1/R=3/8

Cross multiply

1x8=3xR

8=3R

Divide both sides by 3

8/3=3R/3

8/3=R

R=8/3 or 2 2/3 ohms

Which of the following BEST summarizes the relationship between groups and culture and critical thinking?

Answers

Answer:

Groups and culture helps in influencing our values,ethics and beliefs. This influence should always be questioned through the process of thinking critically.

This best summarizes the relationship between groups and culture and critical thinking.

This is a measure of quantity of matter

Answers

Answer:

Mass

Explanation:

Mass is the measure of amount of matter contained within any substance and hence mass determines the weight. Unit of mass is kilogram as per ISI system of units.  

Mass is measured through a balance. The more is the mass of an object, the more the balance tilts towards the object side.  

Weight is equal to product of mass and the gravitational constant i.e 9.8m/s^2

In order to get going fast, eagles will use a technique called stooping, in which they dive nearly straight down and tuck in their wings to reduce their surface area. While stooping, a 6- kg golden eagle can reach speeds of up to 53 m/s . While golden eagles are not very vocal, they sometimes make a weak, high-pitched sound. Suppose that while traveling at maximum speed, a golden eagle heads directly towards a pigeon while emitting a sound at 1.1 kHz. The emitted sound has a sound intensity level of 30 dB when heard at a distance of 5 m .A) Model this stooping golden eagle as an object moving at terminal velocity. The eagle’s drag coefficient is 0.5 and the density of air is 1.2 kg/m 3 . What is the effective cross-sectional area of the eagle’s body while stooping?B) What is the doppler-shifted frequency that the pigeon will hear coming from the eagle?C) Consider the moment when the pigeon is 5 m away from the eagle. At the pigeon’s position, what is the intensity (in W/m^2 ) of the sound the eagle makes?D) The golden eagle slams into the 250- g pigeon, which is initially moving at 10 m/s in the opposite direction (toward the eagle). The eagle grabs the pigeon in its talons, and they move off together in a perfectly inelastic collision. How fast do they move after the collision?

Answers

Answer:

Check the explanation

Explanation:

Part A

F = CA

this drag force balances the weight = 6X 9.8

so

6X9.8 = 0.5 X A X0.5 X 1.2 X 532

A= 0.069 m2

Part B

here the sorce is moving and the observer is at rest

so f= f(- 1 - 1

f = 1.1X10 343 343 – 53

f' = 1.3 KHz

Part C:

given the intensity = 30 dB

we know that I dB = 10 log (I(W/m2))

so we get I (W/m2) = 1000

Part D : The catch

Given that U1 = 53 M1 = 6 kg

U2 =-10 M2=0.25

V1=V2

now conserving momentum

6 X 53 -0.25 X10 =(6+0.25)V

V= 50.48 m/sec

During the process of mountain building, earthquakes sometimes occur along continental-continental convergent boundaries. Which statement best describes the motion of the plates along these boundaries that causes the earthquakes?

The plates push apart from each other with no subduction.
Subduction occurs with the less-dense plate sinking below the other plate.
Subduction occurs with the more-dense plate sinking below the other plate.
The plates smash together with no subduction.

Answers

Answer:

The correct answer is The plates smash together with no subduction. I just took this on Edge. Glad I could help!

The statement best describes the motion of the plates along these boundaries that causes the earthquakes is that The plates smash together with no subduction

For better understanding let's explain what the answer means

it is said that the meeting point of two tectonic plates will causea convergent plate boundary to be formed. one of the converging plates will move under the other as it is known as subduction

From the above we can therefore say that the answer The statement best describes the motion of the plates along these boundaries that causes the earthquakes is that The plates smash together with no subduction, is correct

Learn more motion of the plates along these boundaries from:

https://brainly.com/question/23731885

Other Questions
Plis help I need help Ashley collects colored golf balls from a miniature golf course. She will randomly select one ball from her collection of 9 blue, 7 magenta, and 2 purple golf balls. Develop a probability model based on theoretical probability. 1. What is the total number of colored golf balls in Ashleys collection?2. What is the sample space?S =3. What is the theoretical probability that each event in thesample space occurs? The Indus valley was invaded from the west by the ______________.a.Harrapansc.Babyloniansb.Aryansd.the Indus valley was not invadedPlease select the best answer from the choices providedABCD PLEASE HELP FAST Which transformation preserves size and shape. Choose ALL correct answers A) Dilations B) Rotations C) Translations D) Reflections Question 2 Which transformations preserve orientation? A) Translations B) Rotations C) Reflections D) Dilations Which of these is true of fetal development by the end of the third trimester?A. The placenta is absorbed by the uterus.B. The fetus can survive outside the uterus.c. All development is complete.D. All growth is complete. The distance from the maximum disturbance to the undisturbed position is the____ of a wave I WILL GIVE YOU BRAINLIEST, AND 30 POINTS SO PLEASE HELP ME, IT'S URGENT! who is alive to make it to Ithaca What is the value of 5x+3 when x = 4? The uniform slender bar of mass m and length l is released from rest in the vertical position and pivots on its square end about the corner at O. (a) If the bar is observed to slip when 30 , find the coefficient of static friction s between the bar and the corner. (b)If the end of the bar is notched so that it cannot slip, find the angle at which contact between the bar and the corner ceases. Which statement most accurately examines how President Obama influenced health care during his first term?He required medical insurance companies to reduce their rates so that more Americans could afford health insurance.He eliminated privatized health insurance and established a socialized health care system run by the government.He established a federal health care mandate that would provide low-income Americans with health care.He extended the reach of Medicaid to millions of Americans to create a free universal health care system. Which governmental policy has been responsible for helping move from command to a mixed market economy China?A. Limiting population growthB. Discouraging ties to the pastC. Establishing collective farmsD. Allowing more economic freedom Find the first five terms of the geometric sequence defined by a (n)=10(.1)^n what is the radius and diameter of a 14cm circle if f(x)=-x^2 and g(x) = -x^2+4x+5 what is the product How Does Currency Affect Trade?LILLE+ Which factor plays the greatest role in determining the climate in a highland region? proximity to large bodies of water elevation latitude ocean currents Can someone write why is is so important for teens to get more sleep than the average adult? One paragraph please!! Another khan academy question The probability of event A is 0.5 and probability of event B is 0.2. Given that A and B are independent, then the probability of A and B (A intersection B) is:A) 2.5%B)7 %C)10%D) 14%